Is this sequence arithmetic, geometric, or neither?

Click For Summary
The sequence \(\frac{1}{1}, \frac{1}{2}, \frac{1}{3}, \frac{1}{4}, \ldots, \frac{1}{n}\) is neither arithmetic nor geometric, as the common differences and common ratios are not constant. The correct representation of the sequence is through the sigma notation \(\sum_{k=1}^{n} \frac{1}{k}\), which defines it as a harmonic sequence. A clarification was made regarding the notation, emphasizing that \(k\) should be the variable in the sum, not \(n\). The discussion also highlighted the importance of accurate notation to avoid misinterpretation in mathematical expressions. Overall, the sequence is identified as a harmonic series rather than a simple sequence.
Lebombo
Messages
144
Reaction score
0

Homework Statement



Is the sequence \frac{1}{1}, \frac{1}{2}, \frac{1}{3} , \frac{1}{4}...\frac{1}{n} arithmetic or geometric?

Homework Equations



Common difference and Common ratio formulas

The Attempt at a Solution



I found the common difference from a_{2} - a_{1} =d_{1} and common difference from a_{3} - a_{2} =d_{2}. Since d_{1}≠ d_{2} , then this sequence is not arithmetic.

I did the same thing for the common ratio and found r_{1}≠ r_{2}. So this sequence is not geometric either.

It is simply a sequence defined by the sigma notation \sum_{k=1}^{n} \frac{1}{n}P.S. I found these topics in an Algebra book, but the topic of sequences and series are also present in my calculus book. To moderators, free to move this to the Algebra section if it's felt that this topic would fit better there.
 
Last edited:
Physics news on Phys.org
Much appreciated.
 
Lebombo said:
It is simply a sequence defined by the sigma notation \sum_{k=1}^{n} \frac{1}{n}

Your expression is incorrect. It should be

$$\sum_{k=1}^n \frac{1}{k}.$$

The variable ##k## is what is being summed over, so it must appear in the sum. Here, n is the upper limit.

As written, \sum_{k=1}^{n} \frac{1}{n} is interpreted as ##\frac{1}{n}\sum_{k=1}^n 1 = \frac{1}{n}n = 1##.

You made a similar mistake when writing the sum in your other thread. Be careful, otherwise your equations could easily be misinterpreted.
 
Thanks, although I'm still not 100% proficient on the sigma notation, that was a definite typo as opposed to not having the knowledge in a previous thread. Appreciate the correction.
 
\sum_{k=1}\frac{1}{k}
is a series. You said this was a sequence which would be denoted as
\left\{\frac{1}{k}\right\}_{k=1}^n
 
Question: A clock's minute hand has length 4 and its hour hand has length 3. What is the distance between the tips at the moment when it is increasing most rapidly?(Putnam Exam Question) Answer: Making assumption that both the hands moves at constant angular velocities, the answer is ## \sqrt{7} .## But don't you think this assumption is somewhat doubtful and wrong?

Similar threads

  • · Replies 1 ·
Replies
1
Views
2K
  • · Replies 51 ·
2
Replies
51
Views
5K
  • · Replies 3 ·
Replies
3
Views
1K
  • · Replies 9 ·
Replies
9
Views
2K
  • · Replies 1 ·
Replies
1
Views
1K
  • · Replies 4 ·
Replies
4
Views
3K
  • · Replies 4 ·
Replies
4
Views
2K
Replies
2
Views
2K
  • · Replies 17 ·
Replies
17
Views
2K
  • · Replies 1 ·
Replies
1
Views
1K